Consultant: If Whalley sticks with her current platform in the upcoming election, then she will lose to her opponent ...

c0cald01 on December 9, 2021

Question

Could you explain why D is the Right answer? What was the new concept or idea?

Reply
Create a free account to read and take part in forum discussions.

Already have an account? log in

Jay-Etter on January 14, 2022

In understanding this question, let's first start by reviewing the question type. For strengthen with sufficient premise questions (indicated by question stems such as "the conclusion follows logically if which one of the following is assumed), we are looking for an answer option that will fill any gaps in the reasoning and 100% guarantee the conclusion.

Here the stimulus is telling us that if Whalley sticks with the platform she will lose by a few percentage points among voters under 50, while winning by a larger percentage among voters 50 and over. Imagine Whalley will win only 48% of the vote for voters under 50, but will win 75% of the vote among voters 50 and over. So what do we need to guarantee that Whalley will win the argument? We need to know how many voters there actually are in each group! If there were 100,000 voters 50 and under, and only 100 voters over 50, for example, then Whalley would lose with the current setup. However, if there are more voters 50+ than under 50, because Whalley is winning a larger percentage of the larger voting group in this case, she is numerically sure to win the election. This is exactly what D is telling us, so D is the correct answer here. The other answer options would not allow us to guarantee that Whalley will win the election as long as she doesn't change her platform.